Đến nội dung

Hình ảnh

TOPIC thảo luận, trao đổi toán thi học sinh giỏi khối 10,11 .

hình học tổ hợp số học thpt toán thi hsg vmf hệ phương trình phương trình bất đẳng thức

  • Please log in to reply
Chủ đề này có 81 trả lời

#41
Duy Thai2002

Duy Thai2002

    Sĩ quan

  • Thành viên
  • 433 Bài viết

Bài 17: Tìm tất cả các hàm số $f:\mathbb{N}^*\rightarrow \mathbb{N}^*$ thỏa mãn: $f(mn)+f(m+n)=f(m)f(n)+1,\forall m,n\in \mathbb{N}^*$(*)

Đặt $f(1)=x$

 Trong (*) thay m=n=1 , ta được:

$f(1)+f(2)=f(1)^{2}+1$

$=> f(2)=f(1)^{2}-f(1)+1=x^{2}-x+1$

Trong (*) thay m=2, n=1, ta được:

$f(2)+f(3)=f(1)f(2)+1$

$=> f(3)=f(2)(f(1)-1)+1=(x^{2}-x+1)(x-1)+1$

Trong (*) thay m=3, n=1, ta được:

$ f(3)+f(4)=f(3)f(1)+1 => f(4)=f(3)(f(1)-1)+1=(x^{2}-x+1)(x-1)^{2}+(x-1)+1$

Bằng việc qui nạp m , ta chứng minh được: $ f(m)=(x^{2}-x+1)(x-1)^{m-2}+\sum_{i=1}^{m-3}(x-1)+1$ ($ m>2$)

Trong (*), Thay m=2,n=3, ta được:

$f(6)+f(5)=f(2)f(3)+1$

Theo công thức trên, ta được:

$(x^{2}-x+1)(x-1)^{4}+\sum_{i=1}^{3}(x-1)^{i}+1+(x^{2}-x+1)(x-1)^{3}+\sum_{k=1}^{2}(x-1)^{k}+1=(x^{2}-x+1)((x^{2}-x+1)(x-1)+1)+1$

Giải pt trên, chú ý rằng $x\in N^{*}$ ta được $\begin{bmatrix}x=1 & \\x=2 & \end{bmatrix}$

Với x=1

$=> f(1)=1, f(2)=1, f(m)=(1^{2}-1+1)(1-1)^{m-2}+\sum_{i=1}^{m-3}(1-1)^{i}+1=1$

$=> f(m)=1 \forall m\in N^{*}$

Thử lại, VT(*)=VP(*)=2 nên nhận kết quả trên

Với x=2

$f(1)=2,f(2)=3,f(m)=(2^{2}-2+1)(2-1)^{m-2}+\sum_{i=1}^{m-3}(2-1)^{i}+1=m+1$

$=> f(m)=m+1 \forall m\in N^{*}$

Thử lại, VT(*)=VP(*)=$ m+n+mn+2$ nên nhận $\large f(m)=m+1$

Vậy $ f(m)=1, f(m)=m+1$


Bài viết đã được chỉnh sửa nội dung bởi Duy Thai2002: 16-11-2017 - 18:25

Sự khác biệt giữa thiên tài và kẻ ngu dốt là ở chỗ thiên tài luôn có giới hạn.


#42
NHoang1608

NHoang1608

    Sĩ quan

  • Thành viên
  • 375 Bài viết

Bài 22: Chứng minh rằng tồn tại đa thức $P(x)$ với các hệ số nguyên và không có nghiệm nguyên thỏa mãn

           Với mỗi số nguyên dương $n$ thì luôn tồn tại $x \in \mathbb{Z}$ sao cho $n \mid P(x)$.

 

Turn on với 1 bài số hay :)


Bài viết đã được chỉnh sửa nội dung bởi NHoang1608: 17-11-2017 - 15:18

The greatest danger for most of us is not that our aim is too high and we miss it, but that it is too low and we reach it.

----- Michelangelo----


#43
DinhXuanHung CQB

DinhXuanHung CQB

    Trung sĩ

  • Thành viên
  • 118 Bài viết

Bài 23 <Trích đề HSG 11 2017 Quãng Bình>

Tính giới hạn : $I=\lim_{x->0}\frac{\sqrt{1.2x+1}.\sqrt[3]{2.3x+1}.\sqrt[4]{3.4x+1}...\sqrt[2017]{2016.2017x+1}-1}{x}$

 

Cùng nhau khởi động lại topic nhé :)


Little Homie


#44
viet9a14124869

viet9a14124869

    Trung úy

  • Thành viên
  • 903 Bài viết

Xin lỗi các mem vì thời gian qua mình bận 1 vài việc nên thỉnh thoảng có lên diễn đàn cũng không giữ lửa cho topic được :)) 

Bài 18 [Sưu tầm]: Cho $0<a<b<c$ thỏa mãn $a+b+c=6$ và $ab+bc+ca=9$. Chứng minh rằng: $0<a<1<b<3<c<4$

 

Bài này mình thấy nó khá giống bài British MO trong sách , mình sẽ trình bày theo cách trong sách này , ai có lời giải mới đẹp hơn thì up sớm để mọi người tham khảo nhé :)  có nhiều bài còn dư mong mọi người làm nốt để mình đăng thêm bài mới :)

Đặt p=abc ,xét hàm f(x) =(x-a)(x-b)(x-c) =$x^3-6x^2+9x-p$ 

                                f'(x) = $3x^2-12x+9=3(x-1)(x-3)$ 

Vậy f'(x) có hai nghiệm là x=1 và x=3 . Do f(x) có 3 nghiệm a<b<c nên 1<b<3 và f(1).f(3) <0 

Mặt khác f(1) =4-p  , f(3) =-p  , f(0) =-p  , f(4) =4-p 

Do đó 0<p<4 , suy ra f(0) <0 và f(4) >0 . Vậy 0<a<1<b<3<c<4 . 

 

P/S : Trong hôm nay nếu chưa ai có lời giải cho những bài còn lại thì mong các thành viên đã đề xuất bài toán đó đăng lời giải để mọi người tham khảo nhé ....... :D


Bài viết đã được chỉnh sửa nội dung bởi viet9a14124869: 26-11-2017 - 07:11

                                                                    SÓNG BẮT ĐẦU TỪ GIÓ

                                                                    GIÓ BẮT ĐẦU TỪ ĐÂU ?

                                                                    ANH CŨNG KHÔNG BIẾT NỮA 

                                                                    KHI NÀO...? TA YÊU NHAU .


#45
NHoang1608

NHoang1608

    Sĩ quan

  • Thành viên
  • 375 Bài viết

Lời giải bài 22: 

Xét đa thức bậc 2 $f(x)= (2x-1)(3x-1)$ 

 Nhận thấy rằng với mỗi số nguyên dương $n$ thì ta luôn có $n= 2^{k}.t$ với $k \in \mathbb{N}$ và $t$ lẻ.

Nếu $k=0$ thì $(2;n)=1$ suy ra tồn tại số nguyên $x$ sao cho $2x \equiv 1 (mod n)$ ($x$ là nghịch đảo của 2 modulo n)

 Suy ra tồn tại $x$ nguyên để $n \mid 2x-1$ hay $n \mid (2x-1)(3x-1) = f(x)$.

Nếu $k \geq 1$ thì $(3;2^{k})=1 \Rightarrow \exists a \in \mathbb{Z}: 3a \equiv 1 (mod 2^{k} ).$

                          $(2;t)=1 \Rightarrow \exists b \in \mathbb{Z} : 2b \equiv 1 (mod t)$

Theo định lí thặng dư Trung Hoa và $(2^{k};t)=1$ thì $\exists x \in \mathbb{Z} $ thỏa mãn:

                                                                               $x \equiv a (mod 2^{k})$

                                                                               $x \equiv b (mod t)$

 Hay $\exists x \in \mathbb{Z} : 2^{k} \mid 2x-1, t \mid 3x-1 \Rightarrow n \mid f(x)$                                                                         

Tóm lại với mọi số nguyên dương $n$ thì luôn tồn tại số nguyên $x$ để $n \mid f(x)$


Bài viết đã được chỉnh sửa nội dung bởi NHoang1608: 26-11-2017 - 09:36

The greatest danger for most of us is not that our aim is too high and we miss it, but that it is too low and we reach it.

----- Michelangelo----


#46
NHoang1608

NHoang1608

    Sĩ quan

  • Thành viên
  • 375 Bài viết

Bài toán 24: Kí hiệu $S(n)$ là tổng các chữ số của $n$. Hãy tìm giá trị nhỏ nhất của $S(n)$ khi cho $n$ chạy trên các bội của $2003$.


The greatest danger for most of us is not that our aim is too high and we miss it, but that it is too low and we reach it.

----- Michelangelo----


#47
toannguyenebolala

toannguyenebolala

    Sĩ quan

  • Thành viên
  • 432 Bài viết

Bài toán 25: Cho $a,b,c$ là các hằng số thực và $P(x)=ax^3+bx^2+cx$. Tìm tất cả các số $a,b,c$ sao cho $P(2)=26$ và $\left | P(x) \right |\leq 1$ với mọi số thực $x$ sao cho $\left | x \right |\leq 1$.

Bài toán 26: Xét $k$ là số nguyên dương thỏa mãn tính chất: Tồn tại 2017 tập con $A_{1},A_{2},...A_{2017}$ của tập $\left \{ 0,1,...,10^{2017}-1 \right \}$ (không nhất thiết phân biệt) sao cho mỗi tập có đúng $k$ phần tử và mỗi phần tử của tập $\left \{ 0,1,...,10^{2017}-1 \right \}$ đều biểu diễn được dưới dạng $x_{1}+x_{2}+...+x_{2017}$ trong đó $x_{i}\in A_{i}$ với $i=1,2,...,2017$. Hãy xác định giá trị bé nhất của $k$.


"Đừng khóc, Alfred! Anh cần có đủ nghị lực để chết ở tuổi hai mươi"


#48
slenderman123

slenderman123

    Trung sĩ

  • Thành viên
  • 175 Bài viết

Bài 27:IMO 2013: Vì sợ bài trôi nên mình sẽ giải luôn, bạn nào thích tìm tòi thì hãy tìm nhiều cách giải hơn nhé :))

P/S: Mức tỉnh thì có thể ngang thế thôi ~~

Hình gửi kèm

  • latex.png

Bài viết đã được chỉnh sửa nội dung bởi slenderman123: 30-11-2017 - 20:57

Nguyễn Văn Tự Cường - Trường THPT Chuyên LQĐ - Quảng Trị


#49
cristianoronaldo

cristianoronaldo

    Thượng sĩ

  • Thành viên
  • 233 Bài viết

Bài 27:( cristianoronaldo )

Cho a,b,c,d là các số thực dương thỏa mãn $\dfrac{1}{a}+\dfrac{1}{b}+\dfrac{1}{c}+\dfrac{1}{d}=4$. 

Tìm GTNN của biểu thức:

$P=\dfrac{a+b+c+d}{\sqrt[4]{abcd}}+\dfrac{4}{1+\sqrt[4]{abcd}}$


Bài viết đã được chỉnh sửa nội dung bởi cristianoronaldo: 30-11-2017 - 20:57

Nothing in your eyes


#50
slenderman123

slenderman123

    Trung sĩ

  • Thành viên
  • 175 Bài viết

Lời giải bài 9: Old Memories ~

Hình gửi kèm

  • zubeo.png

Bài viết đã được chỉnh sửa nội dung bởi slenderman123: 30-11-2017 - 21:06

Nguyễn Văn Tự Cường - Trường THPT Chuyên LQĐ - Quảng Trị


#51
slenderman123

slenderman123

    Trung sĩ

  • Thành viên
  • 175 Bài viết

Bài 27:( cristianoronaldo )

Cho a,b,c,d là các số thực dương thỏa mãn $\dfrac{1}{a}+\dfrac{1}{b}+\dfrac{1}{c}+\dfrac{1}{d}=4$. 

Tìm GTNN của biểu thức:

$P=\dfrac{a+b+c+d}{\sqrt[4]{abcd}}+\dfrac{4}{1+\sqrt[4]{abcd}}$

$4=\sum \frac{1}{a}\geq \frac{1}{\sqrt[4]{abcd}}\Rightarrow \sqrt[4]{abcd}\leq 1$

Suy ra $\frac{a+b+c+d}{\sqrt[4]{abcd}}+\frac{4}{1+\sqrt[4]{abcd}}\geq 6$

P/S: Liệu chỉ thế này hay mình đã sai :))


Nguyễn Văn Tự Cường - Trường THPT Chuyên LQĐ - Quảng Trị


#52
viet9a14124869

viet9a14124869

    Trung úy

  • Thành viên
  • 903 Bài viết

$4=\sum \frac{1}{a}\geq \frac{1}{\sqrt[4]{abcd}}\Rightarrow \sqrt[4]{abcd}\leq 1$

Suy ra $\frac{a+b+c+d}{\sqrt[4]{abcd}}+\frac{4}{1+\sqrt[4]{abcd}}\geq 6$

P/S: Liệu chỉ thế này hay mình đã sai :))

Sai rồi , đoạn bôi đỏ bạn làm sai rồi nhé ....xem lại đi . 

 

P/S: Các trang trước  còn nhiều bài hay mà mình cũng không muốn bỏ , bạn nào giải được thì đăng lên cho mọi người xem với nhé .... :D  


                                                                    SÓNG BẮT ĐẦU TỪ GIÓ

                                                                    GIÓ BẮT ĐẦU TỪ ĐÂU ?

                                                                    ANH CŨNG KHÔNG BIẾT NỮA 

                                                                    KHI NÀO...? TA YÊU NHAU .


#53
NMD202

NMD202

    Binh nhất

  • Thành viên mới
  • 37 Bài viết

Bài 28[sáng tác] Cho $a,b,c,d$ là các số nguyên a khác 0. Giả sử đa thức$f(x)=ax^2+bx+c$ thỏa mãn $f_{2017}(d)=d$. Chứng minh $f(d)=d$

Bài 29[phỏng theo Korea 1997] Chứng minh rằng với $k \geq 2017, k \in Z$, luôn tồn tại hai dãy số $(a_{i})_{i=1}^{k}$ và $(b_{i})_{i=1}^{k}$ thỏa mãn 
(a) $a_{i}, b_{i}$ $\in$ ${1,2016,2016^2,...,}$ với $i=1,2,..,k$

(b) $a_{i} \neq b_{i}$ với $i=1,2,..,k$

(c) $a_{i} \leq a_{i+1}$ và  $b_{i} \leq b_{i+1}$ với $i=1,2,..,k-1$

(d) $\Sigma_{i=1}^{k}a_{i}=\Sigma_{i=1}^{k}b_{i}$

 


Bài viết đã được chỉnh sửa nội dung bởi NMD202: 04-12-2017 - 01:40

@NguyenMinhDuy - frTK19.LQĐ.BĐ 

Bài hình CĐT LQĐ Bình Định  https://diendantoanh...ường-thẳng-qua/


#54
NMD202

NMD202

    Binh nhất

  • Thành viên mới
  • 37 Bài viết

Bài 28: Ta có $f(d)-d=f_{2018}(d)-f_{2017}(d)=f(f_{2017}(d))-f(f_{2016}(d)) \vdots f_{2017}(d)-f_{2016}(d),$
$f_{2017}(d)-f_{2016}(d) = f(f_{2016}(d))-f(f_{2015}(d)) \vdots f_{2016}(d)-f_{2015}(d),$
Thực hiện tương tự, ta được :
$f_{2}(d)-f_{1}(d) = f(f(d))-f(d) \vdots f(d)-d.$

Từ đó, $\left | f(d)-d \right |=\left | f_{2}(d)-f(d) \right |=...=\left | f_{2017}(d)-f_{2016}(d) \right |$

Vì $0=f_{2017}(d)-d=\left\{\begin{matrix} k \\ 2017k \end{matrix}\right.$ nên k=0 suy ra $f(d)=d$

 

Bài 29:

Với $k \leq 2016$ giả sử tồn tại hai dãy $(a_{i})$ và $(b_{i})$ thỏa mãn đề bài. 
Khi đó do $a_{1} \neq b_{1}$ nên KMTTQ có thể giả sử $a_{1} < b_{1}$.
Từ điều kiện (a), tồn tại các số nguyên $0 \leq m < n$ sao cho: $a_{1}=2016^m,  b_{1}=2016^n$

Do  $b_{i} \geq  b_{1}$ với mọi $i$ nên tổng $\sum _{i=1}^{k}b_{i} \vdots 2016^n$
Gọi $l$ là số các chỉ số i sao cho $a_{i} =  a_{1} = 2016^m$, khi đó 
$\sum _{i=1}^{k}a_{i} \vdots 2016^n \equiv l.2016^m \equiv 0 (mod2016^n)$ 
Vì $n \geq m+1$ nên $l \vdots 2016$, suy ra $k \geq l \geq 2016$
Vậy ta phải có $k=l=2016$. Từ đó suy ra 

$\sum _{i=1}^{k}a_{i} = \sum _{i=1}^{2016}a_{i}  = 2016^{m+1}= \sum _{i=1}^{2016}b_{i} > 2016.b_{1}=2016^{n+1} > 2016^{m+1}$ (vô lý)

Suy ra dpcm


@NguyenMinhDuy - frTK19.LQĐ.BĐ 

Bài hình CĐT LQĐ Bình Định  https://diendantoanh...ường-thẳng-qua/


#55
slenderman123

slenderman123

    Trung sĩ

  • Thành viên
  • 175 Bài viết

Thấy TOPIC hơi vắng bóng dãy số nhỉ :))

Mình sẽ lấy ví dụ của một số bài dãy số:

Trước hết là một bài dãy số cấp số nhân:

Cho dãy $(F_{n})$ xác định bởi

$\left\{\begin{matrix} F_{1}=1,F_{2}=1\\ F_{n+2}=F_{n+1}+F_{n} \end{matrix}\right.$

Tìm CTTQ.

Phương trình đặc trưng: $x^{2}-x-1=0\Rightarrow \left\{\begin{matrix} x_{1}=\frac{1+\sqrt{5}}{2}\\ x_{2}=\frac{1-\sqrt{5}}{2} \end{matrix}\right.$

Suy ra $\left\{\begin{matrix} F_{1}=e_{1}.\frac{1+\sqrt{5}}{2}+e_{2}.\frac{1-\sqrt{5}}{2}=1\\ F_{2}=e_{1}.(\frac{1+\sqrt{5}}{2})^{2}+e_{2}.(\frac{1-\sqrt{5}}{2})^{2}=1 \end{matrix}\right. \Rightarrow$ $\left\{\begin{matrix} e_{1}=\frac{1}{\sqrt{5}}\\ e_{2}=-\frac{1}{\sqrt{5}} \end{matrix}\right.$

Suy ra $F_{n}=\frac{1}{\sqrt{5}}(\frac{1+\sqrt{5}}{2})^{n}-\frac{1}{\sqrt{5}}(\frac{1-\sqrt{5}}{2})^{n}$

Tiếp theo mình sẽ lấy một ví dụ về dãy cấp số cộng:

Cho $(F_{n})$ xác định bởi:$\left\{\begin{matrix} u_{1}=2\\ u_{n}=2u_{n-1}+n-2 \end{matrix}\right.$

Đặt $u_{n}=v_{n}+f(n) $ sao cho $v_{n}=2v_{n-1}$

Ta có: $f(n)=2f(n-1)+n-2$

Đến giờ chọn $f(n)=an+b$ thì ta có: $an+b=2a(n-1)+2b+n-2\Rightarrow an+b=2an-2a+2b+n-2\Rightarrow \left\{\begin{matrix} a=2a+1\\ b=2b-2a-2 \end{matrix}\right.\Rightarrow \left\{\begin{matrix} a=-1\\ b=-1 \end{matrix}\right.$

Suy ra $f(n)=-n-1\Rightarrow v_{1}=u_{1}-f(1)=4$ $v_{n}=2v_{n-1}=2^{n-1}.v_{1}=4.2^{n-1}=2^{n+1}\Rightarrow u_{n}=2^{n-1}-n-1$

Mong từ đây, các bạn hãy đề nghị các bài tập về dãy số đi ạ :P


Bài viết đã được chỉnh sửa nội dung bởi slenderman123: 09-12-2017 - 16:44

Nguyễn Văn Tự Cường - Trường THPT Chuyên LQĐ - Quảng Trị


#56
viet9a14124869

viet9a14124869

    Trung úy

  • Thành viên
  • 903 Bài viết

Khởi động lại topic nhé mọi người :) 

Bài toán số 30 (sưu tầm ) : Cho điểm P cố định nằm trong đường tròn (O;R ) và hai điểm A,B chạy trên đường tròn đó sao cho $\widehat{APB}=90^o$ . Gọi M là trung điêm của dây AB và H là hình chiếu của P xuống AB . Chứng minh M,H luôn cùng thuộc một đường tròn cố định . 

Bài toán số 31 (sưu tầm ) : Cho tam giác ABC , Tìm quỹ tích điểm M thỏa mãn $MA^2+2MB^2-3MC^2=k$ ( k là số thực tùy ý ) 

 

P/S : Hai bài toán trên mình lấy trong mục tích vô hướng của các vector , mọi người tham gia giải nhé ... 

 

_______________________________________________________________________________________________

 

Mình sẽ tiếp thêm 1 bài đại số : 

Bài toán số 32 (sưu tầm ) : Cho hàm số $y=\left | \sqrt[4]{11+4x-x^4}-2m^2+5m-1 \right |$ . Tìm m để giá trị lớn nhất của hàm số là nhỏ nhất . 

 

                                            


                                                                    SÓNG BẮT ĐẦU TỪ GIÓ

                                                                    GIÓ BẮT ĐẦU TỪ ĐÂU ?

                                                                    ANH CŨNG KHÔNG BIẾT NỮA 

                                                                    KHI NÀO...? TA YÊU NHAU .


#57
trieutuyennham

trieutuyennham

    Sĩ quan

  • Thành viên
  • 470 Bài viết

Bài toán số 31 (sưu tầm ) : Cho tam giác ABC , Tìm quỹ tích điểm M thỏa mãn $MA^2+2MB^2-3MC^2=k$ ( k là số thực tùy ý ) 

Gọi $I$ là điểm thỏa mãn $\overrightarrow{IA}+2\overrightarrow{IB}-3\overrightarrow{IC}=\overrightarrow{0}$

Ta có

$VT=(\overrightarrow{MI}+\overrightarrow{IA})^2+(\overrightarrow{MB}+2\overrightarrow{IB})^2-3(\overrightarrow{MC}+\overrightarrow{IC})^2$

=$IA^2+2IB^2-3IC^2=k$


Bài viết đã được chỉnh sửa nội dung bởi trieutuyennham: 14-12-2017 - 21:23


#58
gosh

gosh

    Lính mới

  • Thành viên mới
  • 8 Bài viết

Bài 33 (sưu tầm): Tìm n là số nguyên dương, p là số nguyên tố, n $\leq$ 2p sao cho 

 

 $\left ( p-1 \right )^{n}+1 \vdots n^{p-1}$

Bài viết đã được chỉnh sửa nội dung bởi gosh: 15-12-2017 - 00:05

mh.


#59
royal1534

royal1534

    Trung úy

  • Điều hành viên THCS
  • 773 Bài viết

 

Bài 33 (sưu tầm): Tìm n là số nguyên dương, p là số nguyên tố, n $\leq$ 2p sao cho 

 

 $\left ( p-1 \right )^{n}+1 \vdots n^{p-1}$

 

Lời giải : 

Bổ đề (quen thuộc).
Cho $a,n$ nguyên dương. Nếu tồn tại một số nguyên tố $q$ thỏa $q | a^n+1$ và $(q-1,n)=1$ thì $q | a+1$.
Trở lại bài toán : 
Gọi $q$ là ước nguyên tố nhỏ nhất của $n$. Khi đó $(q-1,n)=1$

Suy ra $q | (p-1)+1 \Rightarrow q=1$ hoặc $q=p$ 
Xét $q=1$ thì suy ra $n=1$. Dẫn tới mọi cặp $(n,p)=(1,p)$ đều thỏa mãn
Xét $q=p$.

Suy ra $p^{p-1} \mid n^{p-1} \mid (p-1)^n+1$

Áp dụng bổ đề LTE

$\Rightarrow v_{p}(p-1+1)+v_{p}(n) \geq p-1$

$\Rightarrow v_{p}(n) \geq p-2 $

$\Rightarrow p^{p-2} \mid n$

Mà $n \leq 2p \Rightarrow p=2; 3 \Rightarrow n=2; 3$


Bài viết đã được chỉnh sửa nội dung bởi royal1534: 17-12-2017 - 23:04


#60
royal1534

royal1534

    Trung úy

  • Điều hành viên THCS
  • 773 Bài viết

Bài toán 34 : Cho $F_{n}=2^{2^n}+1$ là số Fermat thứ $n$.

a, Với mỗi $n$ nguyên dương. Gọi $p$ là ước nguyên tố lẻ bất kì của $F_{n}$. Chứng minh $p \equiv 1$ (mod $2^{n+2}$).

b, Chứng minh tồn tại vô số số nguyên tố $p,q$ phân biệt thỏa $q \mid 2^{p-1}-1$ và $p \mid 2^{q-1}-1$.


Bài viết đã được chỉnh sửa nội dung bởi royal1534: 17-12-2017 - 23:04






Được gắn nhãn với một hoặc nhiều trong số những từ khóa sau: hình học, tổ hợp, số học, thpt, toán thi hsg, vmf, hệ phương trình, phương trình, bất đẳng thức

1 người đang xem chủ đề

0 thành viên, 1 khách, 0 thành viên ẩn danh